Difference between revisions of "1967 IMO Problems/Problem 1"

m
m
Line 5: Line 5:
  
 
----
 
----
 +
 
==Solution==
 
==Solution==
 
To start our proof we draw a parallelogram with the requested sides. We notice that by drawing the circles with centers A, B, C, D that the length of <math>a</math> must not exceed 2 (the radius for each circle) or the circles will not meet and thus not cover the parallelogram.
 
To start our proof we draw a parallelogram with the requested sides. We notice that by drawing the circles with centers A, B, C, D that the length of <math>a</math> must not exceed 2 (the radius for each circle) or the circles will not meet and thus not cover the parallelogram.
Line 30: Line 31:
  
 
--[[User:Bjarnidk|Bjarnidk]] 02:16, 17 May 2013 (EDT)
 
--[[User:Bjarnidk|Bjarnidk]] 02:16, 17 May 2013 (EDT)
 +
 +
 +
==Remarks (added by pf02, September 2024)==
 +
 +
1. I am sorry to be so harshly critical, but the solution above
 +
is deeply flawed.  Not only it has errors, but the logic is flawed.
 +
 +
It shows that when <math>a = 2, \alpha = \frac{\pi}{3}</math> the parallelogram
 +
is covered by the circles of radius <math>1</math> centered at <math>A, B, C, D</math>, and
 +
the inequality in the problem is true.  (Even this is incomplete, while
 +
giving too many, unnecessary details.)  (Note that this is not a case
 +
which satisfies the conditions of the problem since <math>\triangle ABD</math> is
 +
right, not acute.)
 +
 +
In the last two lines it gives some reasoning about other values of
 +
<math>\alpha</math> which is incomprehensible to this reader.
 +
 +
In one short sentence: this is not a solution.
 +
 +
2. The problem itself is mildly flawed.  To see this, denote <math>S1, S2</math>
 +
the following two statements:
 +
 +
S1: The parallelogram <math>ABCD</math> is covered by the four circles of radius
 +
<math>1</math> centered at <math>A, B, C, D</math>.
 +
 +
S2: We have <math>a \le \cos \alpha + \sqrt{3} \sin \alpha</math>.
 +
 +
The problem says that if <math>\triangle ABD</math> is acute, <math>S1</math> and <math>S2</math> are
 +
equivalent, i.e. they imply each other.
 +
 +
Notice that <math>S2</math> can be rewritten as
 +
<math>a \le 2 \cos \left( \alpha - \frac{\sqrt{\pi}}{3} \right)</math>.
 +
 +
Now notice that if <math>a \le 1</math> then S1 is obviously true.
 +
 +
Also, notice that if <math>a \le 1</math> and
 +
<math>\alpha \in \left( 0, \frac{\sqrt{\pi}}{2} \right) </math>
 +
then <math>S2</math> as true.  Indeed
 +
<math>\left( \alpha - \frac{\sqrt{\pi}}{3} \right) \in
 +
\left( -\frac{\sqrt{\pi}}{3}, \frac{\sqrt{\pi}}{6} \right)</math>, so
 +
<math>\cos</math> is <math>> \frac{1}{2}</math> on this interval, so the right hand side
 +
of <math>S2</math> is <math>> 1 \ge a</math>.
 +
 +
We see that if <math>a \le 1</math> and <math>\triangle ABD</math> is acute, both <math>S1</math>
 +
and <math>S2</math> are true.  We can not say that one implies the other in
 +
the usual meaning of the word "imply": the two statements just
 +
happen to be both true.
 +
 +
If we take <math>a > 1</math> then the problem is a genuine problem, and
 +
there is something to prove.
 +
 +
 +
 +
 +
 +
 +
 +
TO BE CONTINUED.  I AM SAVING MID WAY SO AS NOT TO LOSE WORK DONE SO FAR.
 +
 
-----
 
-----
Solution can also be found here [https://artofproblemsolving.com/community/c6h21154p137323]
+
 
 +
A solution can also be found here [https://artofproblemsolving.com/community/c6h21154p137323]
  
 
== See Also == {{IMO box|year=1967|before=First question|num-a=2}}
 
== See Also == {{IMO box|year=1967|before=First question|num-a=2}}

Revision as of 13:37, 17 September 2024

Let $ABCD$ be a parallelogram with side lengths $AB = a$, $AD = 1$, and with $\angle BAD = \alpha$. If $\Delta ABD$ is acute, prove that the four circles of radius $1$ with centers $A$, $B$, $C$, $D$ cover the parallelogram if and only if

$a\leq \cos \alpha+\sqrt{3}\sin \alpha$ $\ \ \ \ \ \ \ \ \ \ (1)$


Solution

To start our proof we draw a parallelogram with the requested sides. We notice that by drawing the circles with centers A, B, C, D that the length of $a$ must not exceed 2 (the radius for each circle) or the circles will not meet and thus not cover the parallelogram.

To prove our conjecture we draw a parallelogram with $a=2$ and draw a segment $DB$ so that $\angle ADB=90^{\circ}$

This is the parallelogram which we claim has the maximum length on $a$ and the highest value on any one angle.

We now have two triangles inside a parallelogram with lengths $1, 2$ and $x$, $x$ being segment $DB$. Using the Pythagorean theorem we conclude:

$1^2+x^2=2^2\\x=\sqrt{3}$

Using trigonometric functions we can compute:

$cos\alpha=\frac{1}{2}\\sin\alpha=\frac{\sqrt{3}}{2}$

Notice that by applying the $arcsine$ and $arccos$ functions, we can conclude that our angle $\alpha=60^{\circ}$

To conclude our proof we make sure that our values match the required values for maximum length of $a$

$a\leq\cos\alpha+\sqrt{3}\sin\alpha\\\\a\leq\frac{1}{2}+\sqrt{3}\cdot \frac{\sqrt{3}}{2}\\\\a\leq 2$

Notice that as $\angle\alpha$ decreases, the value of (1) increases beyond 2. We can prove this using the law of sines. Similarly as $\angle\alpha$ increases, the value of (1) decreases below 2, confirming that (1) is only implied when $\Delta ABD$ is acute.

--Bjarnidk 02:16, 17 May 2013 (EDT)


Remarks (added by pf02, September 2024)

1. I am sorry to be so harshly critical, but the solution above is deeply flawed. Not only it has errors, but the logic is flawed.

It shows that when $a = 2, \alpha = \frac{\pi}{3}$ the parallelogram is covered by the circles of radius $1$ centered at $A, B, C, D$, and the inequality in the problem is true. (Even this is incomplete, while giving too many, unnecessary details.) (Note that this is not a case which satisfies the conditions of the problem since $\triangle ABD$ is right, not acute.)

In the last two lines it gives some reasoning about other values of $\alpha$ which is incomprehensible to this reader.

In one short sentence: this is not a solution.

2. The problem itself is mildly flawed. To see this, denote $S1, S2$ the following two statements:

S1: The parallelogram $ABCD$ is covered by the four circles of radius $1$ centered at $A, B, C, D$.

S2: We have $a \le \cos \alpha + \sqrt{3} \sin \alpha$.

The problem says that if $\triangle ABD$ is acute, $S1$ and $S2$ are equivalent, i.e. they imply each other.

Notice that $S2$ can be rewritten as $a \le 2 \cos \left( \alpha - \frac{\sqrt{\pi}}{3} \right)$.

Now notice that if $a \le 1$ then S1 is obviously true.

Also, notice that if $a \le 1$ and $\alpha \in \left( 0, \frac{\sqrt{\pi}}{2} \right)$ then $S2$ as true. Indeed $\left( \alpha - \frac{\sqrt{\pi}}{3} \right) \in \left( -\frac{\sqrt{\pi}}{3}, \frac{\sqrt{\pi}}{6} \right)$, so $\cos$ is $> \frac{1}{2}$ on this interval, so the right hand side of $S2$ is $> 1 \ge a$.

We see that if $a \le 1$ and $\triangle ABD$ is acute, both $S1$ and $S2$ are true. We can not say that one implies the other in the usual meaning of the word "imply": the two statements just happen to be both true.

If we take $a > 1$ then the problem is a genuine problem, and there is something to prove.




TO BE CONTINUED. I AM SAVING MID WAY SO AS NOT TO LOSE WORK DONE SO FAR.


A solution can also be found here [1]

See Also

1967 IMO (Problems) • Resources
Preceded by
First question
1 2 3 4 5 6 Followed by
Problem 2
All IMO Problems and Solutions